3
$\begingroup$

I have the following #P-complete problem:

Given an alphabet $\Sigma$ and a matrix $M$ where each entry can be a symbol from $\Sigma$ or the wildcard symbol $*$, find the number of strings $s$ with the property that there exists $i$ such that for all $j$, $M_{i,j} \neq *$ implies $s_j = M_{i,j}$.

Example: If $\Sigma = \{a,b,c\}$ and $M$ = $$\left(\begin{matrix} a&*&b \\ *&c&* \end{matrix} \right)$$ then there are three satisfying strings: $aab,abb,acb$.

I was told this can be solved using BDDs, somehow, such that a lot of caching is possible. How does that work? Also, what are other algorithms for this?

Proof of #P-completeness: We reduce from #UNSAT, the problem of counting the number of unsatisfying assignments for a boolean formula. Given a formula $\varphi$ with $n$ variables and $m$ clauses, make an $m$ by $n$ matrix $M$ over the alphabet $\Sigma = \{0,1\}$ such that $$M_{i,j} = \begin{cases} 1\text{ if the $j^{th}$ literal is unnegated in the $i^{th}$ clause,} \\ 0\text{ if the $j^{th}$ literal is negated in the $i^{th}$ clause,} \\ *\text{ otherwise.} \end{cases} $$ Then the number of strings with the above property equals the number of unsatisfying assignments for $\varphi$.

$\endgroup$
2
  • $\begingroup$ I'm sorry, but I don't know what "some $i$ and all $j$ means". Does that mean $\exists i \forall j$? $\forall j \exists i$? The two are not equivalent. I suggest you edit the question to state the property more precisely. $\endgroup$
    – D.W.
    Dec 21, 2015 at 1:34
  • $\begingroup$ @D.W. The former. In other words, the string only has to satisfy one row of the matrix; each row can be thought of as a "clause" and the matrix as a DNF-type formula. That's why it's #P-complete: counting models to DNF formulae is #P-complete, and if you limit the alphabet to {0,1}, then the matrix is exactly like DNF. $\endgroup$ Dec 21, 2015 at 2:00

1 Answer 1

1
$\begingroup$

Yes, you can use a BDD to solve this, but BDD's aren't magic: they don't magically allow you to solve a #P-complete problem efficiently. You can use BDD's to solve this, but the resulting solution might be highly inefficient.

How do you use BDD's to solve this? Broadly, you build a BDD that represents all strings that satisfy the property, and then you count how many strings are accepted by this BDD.

To learn how to build a BDD that accepts all strings that satisfy the property, I suggest that you study basic material on BDDs. The key property that's relevant here is that given BDDs for the sets $S,S'$, it is easy to construct a BDD for the set $S \cup S'$. So, you build a BDD for the set of strings that are accepted by each row of the matrix, and then you take their union to get a BDD for the union. Beware that the resulting BDD could have exponential size.

Once you have a BDD, it's easy to count the number of strings accepted by a BDD. It's an especially simple application of dynamic programming: for each node $n$ in the BDD, you count the number of paths from $n$ to some leaf. These numbers can be filled in by visiting the nodes in bottom-up order, and using dynamic programming.

There's nothing fancy here. Once you are familiar with BDD's, everything here should look bog-standard. So, if this looks mysterious, I recommend you spend some quality time with a textbook that explains how BDD's work.


Other algorithms: you could also use any algorithm for #SAT. There are off-the-shelf algorithms and tools for computing #SAT. It's straightforward to translate your problem to an instance of #SAT, and then apply one of those tools. See, e.g., https://cstheory.stackexchange.com/q/1295/5038, Is there a sometimes-efficient algorithm to solve #SAT?, https://cstheory.stackexchange.com/q/18135/5038.

$\endgroup$
2
  • $\begingroup$ What kinds of sub-trees can you cache? I'm interested in a particular case, I'll edit my question. $\endgroup$ Dec 21, 2015 at 4:37
  • $\begingroup$ @RenéG, if you want to know how to count how many strings are accepted by a BDD, I recommend you spend some time reading about BDD's (e.g., from a textbook), then if you still can't see how to do it, ask a new question... and show what approaches you've considered. $\endgroup$
    – D.W.
    Dec 21, 2015 at 9:38

Your Answer

By clicking “Post Your Answer”, you agree to our terms of service and acknowledge you have read our privacy policy.

Not the answer you're looking for? Browse other questions tagged or ask your own question.